Recent changes
Track the most recent changes to the wiki on this page.
8 November 2024
18:07 | 2024 AMC 10A Problems/Problem 14 (11 changes | history) . . (+595) . . [Cat9897 (4×); Aspalapati75 (7×)] | ||
18:07 (cur | prev) . . (+147) . . Cat9897 (talk | contribs) (→Solution 1) | |||
18:03 (cur | prev) . . (+1) . . Aspalapati75 (talk | contribs) (→Solution 1) | |||
18:03 (cur | prev) . . (0) . . Aspalapati75 (talk | contribs) (→Solution 1) | |||
18:03 (cur | prev) . . (0) . . Aspalapati75 (talk | contribs) (→Solution 1) | |||
18:02 (cur | prev) . . (+66) . . Aspalapati75 (talk | contribs) (→Solution 1) | |||
18:01 (cur | prev) . . (-5) . . Cat9897 (talk | contribs) (→Problem) | |||
18:00 (cur | prev) . . (+280) . . Aspalapati75 (talk | contribs) (→Solution 1) | |||
18:00 (cur | prev) . . (-2) . . Cat9897 (talk | contribs) (→Problem) | |||
m | 18:00 (cur | prev) . . (+5) . . Cat9897 (talk | contribs) (→Problem) | ||
17:57 (cur | prev) . . (+85) . . Aspalapati75 (talk | contribs) (→Solution 1) | |||
17:56 (cur | prev) . . (+18) . . Aspalapati75 (talk | contribs) (→Diagram) |
N 18:07 | 2024 AMC 12A Problems/Problem 19 (4 changes | history) . . (+1,130) . . [Eevee9406; Lptoggled (3×)] | ||
18:07 (cur | prev) . . (+543) . . Eevee9406 (talk | contribs) | |||
18:03 (cur | prev) . . (+1) . . Lptoggled (talk | contribs) (→solution 1) | |||
18:02 (cur | prev) . . (-1) . . Lptoggled (talk | contribs) (→solution 1) | |||
N | 18:02 (cur | prev) . . (+587) . . Lptoggled (talk | contribs) (Created page with "==solution 1== <math>\angle CBA=60 ^\circ</math> by Circle Theorem} Let <math>AC=u</math>, apply cosine law on <math>\triangle ACD</math> <cmath>u^2=3^2+5^2-2(3)(5)cos120</cma...") |
18:07 | AMC historical results (diff | hist) . . (+7) . . Mrinala (talk | contribs) (→AMC 10A: 2024 Fall) |
18:06 | 2024 AMC 12A Problems/Problem 7 (diff | hist) . . (+129) . . Mathcosine (talk | contribs) (→Solution 1 (technical vector bash)) |
18:06 | 2024 AMC 10A Problems/Problem 6 (diff | hist) . . (+1,257) . . World123 (talk | contribs) |
18:05 | 2024 AMC 10A Problems/Problem 21 (12 changes | history) . . (+2,772) . . [Yth; Tacos are yummy 1; Elainegu (10×)] | ||
18:05 (cur | prev) . . (+1,652) . . Yth (talk | contribs) (→Solution) | |||
17:59 (cur | prev) . . (+401) . . Tacos are yummy 1 (talk | contribs) | |||
17:59 (cur | prev) . . (+19) . . Elainegu (talk | contribs) | |||
17:58 (cur | prev) . . (-31) . . Elainegu (talk | contribs) | |||
17:58 (cur | prev) . . (+11) . . Elainegu (talk | contribs) | |||
17:56 (cur | prev) . . (-1) . . Elainegu (talk | contribs) | |||
17:56 (cur | prev) . . (+14) . . Elainegu (talk | contribs) | |||
17:54 (cur | prev) . . (+13) . . Elainegu (talk | contribs) | |||
17:54 (cur | prev) . . (0) . . Elainegu (talk | contribs) | |||
17:52 (cur | prev) . . (+23) . . Elainegu (talk | contribs) | |||
17:52 (cur | prev) . . (+5) . . Elainegu (talk | contribs) | |||
17:51 (cur | prev) . . (+666) . . Elainegu (talk | contribs) |
N 18:05 | 2024 AMC 12A Problems/Problem 15 (6 changes | history) . . (+1,697) . . [Lptoggled; Eevee9406 (2×); Ev2028 (3×)] | ||
18:05 (cur | prev) . . (+393) . . Lptoggled (talk | contribs) (→Solution 1) | |||
m | 18:00 (cur | prev) . . (+166) . . Eevee9406 (talk | contribs) | ||
17:57 (cur | prev) . . (+624) . . Eevee9406 (talk | contribs) | |||
17:56 (cur | prev) . . (-4) . . Ev2028 (talk | contribs) (→Solution) | |||
17:56 (cur | prev) . . (+420) . . Ev2028 (talk | contribs) | |||
N | 17:50 (cur | prev) . . (+98) . . Ev2028 (talk | contribs) (Created page with "The roots of x^3 + 2x^2 − x + 3 are p, q, and r. What is the value of (p^2 + 4)(q^2 + 4)(r^2+4)?") |
m 18:05 | 2012 AMC 10A Problems/Problem 12 (diff | hist) . . (0) . . Mathforce (talk | contribs) (→Solution 2) |
N 18:04 | 2024 AMC 12A Problems/Problem 18 (3 changes | history) . . (+1,976) . . [Eevee9406; Lptoggled (2×)] | ||
18:04 (cur | prev) . . (+1,094) . . Eevee9406 (talk | contribs) | |||
17:57 (cur | prev) . . (+23) . . Lptoggled (talk | contribs) (→Solution 1) | |||
N | 17:55 (cur | prev) . . (+859) . . Lptoggled (talk | contribs) (Created page with "==Solution 1== Let the midpoint of <math>AC</math> be <math>P</math> We see that no matter how many moves we do, <math>P</math> stays where it is Now we can find the angle of...") |
18:03 | 2024 AMC 10A Problems/Problem 16 (3 changes | history) . . (+1,349) . . [I am suk at math 2 (3×)] | ||
18:03 (cur | prev) . . (+125) . . I am suk at math 2 (talk | contribs) (→Solution) | |||
18:01 (cur | prev) . . (+59) . . I am suk at math 2 (talk | contribs) (→Solution) | |||
18:00 (cur | prev) . . (+1,165) . . I am suk at math 2 (talk | contribs) |
m 18:02 | 2024 AMC 10A Problems/Problem 20 (diff | hist) . . (+2) . . Eevee9406 (talk | contribs) |
m 18:01 | 2024 AMC 12A Problems/Problem 16 (diff | hist) . . (+88) . . Eevee9406 (talk | contribs) |
18:00 | 2024 AMC 10A Problems/Problem 15 (3 changes | history) . . (+283) . . [Technodoggo (3×)] | ||
18:00 (cur | prev) . . (-473) . . Technodoggo (talk | contribs) (sol 2's process is slightly better than 2a's, so combining the justification for 2a and the process for 2 makes sense =D also, I wonder who the reader of this message is! 1434 hehehehhehehhe he he he eh eh eh e) | |||
m | 17:55 (cur | prev) . . (0) . . Technodoggo (talk | contribs) (→Solution 2a (perhaps more clear): oops 3\neq5) | ||
17:54 (cur | prev) . . (+756) . . Technodoggo (talk | contribs) |
17:56 | 2024 AMC 10A Problems/Problem 22 (2 changes | history) . . (+29) . . [Cat9897 (2×)] | ||
17:56 (cur | prev) . . (+8) . . Cat9897 (talk | contribs) (→Solution) | |||
17:49 (cur | prev) . . (+21) . . Cat9897 (talk | contribs) (→Solution) |